Find the limit of $int_{0}^{pi }frac{sin nx}{nx} mathrm dx $.












1














If $a>0,$ show that $lim limits_{n to infty} int_{ a}^{pi} frac{sin nx}{nx} mathrm dx =0,$ what happens if $a=0.$

The first part is easy since the integrand uniformly converges to zero on $[a, pi]$ and for each $n, ; f_n(x)= frac {sin nx}{nx}$ is continuous on $[a,pi].$

But for $a=0$ I don't understand how to proceed. I was thinking of using bounded convergence theorem but that required uniform boundedness of the integrand .
Please give me a hint. Thanks in advance.










share|cite|improve this question
























  • is this what you aimed to write? Please check.
    – user376343
    Nov 23 at 19:34






  • 1




    Where is $a$ in the integral?
    – Will M.
    Nov 23 at 19:34












  • No, sorry, I fixed it
    – Normal
    Nov 23 at 19:38










  • The integrand is continuous and bounded, so what happens when you integrate on $[0,a]$ with $ato0$?
    – Jean-Claude Arbaut
    Nov 23 at 19:45










  • The integrand is continuous on [0,pi] considering $ f_n(0) $=1 and hence it is bounded on [0,pi]. Sorry I am a beginner in real analysis ..I could not understand what you said .
    – Normal
    Nov 23 at 20:17


















1














If $a>0,$ show that $lim limits_{n to infty} int_{ a}^{pi} frac{sin nx}{nx} mathrm dx =0,$ what happens if $a=0.$

The first part is easy since the integrand uniformly converges to zero on $[a, pi]$ and for each $n, ; f_n(x)= frac {sin nx}{nx}$ is continuous on $[a,pi].$

But for $a=0$ I don't understand how to proceed. I was thinking of using bounded convergence theorem but that required uniform boundedness of the integrand .
Please give me a hint. Thanks in advance.










share|cite|improve this question
























  • is this what you aimed to write? Please check.
    – user376343
    Nov 23 at 19:34






  • 1




    Where is $a$ in the integral?
    – Will M.
    Nov 23 at 19:34












  • No, sorry, I fixed it
    – Normal
    Nov 23 at 19:38










  • The integrand is continuous and bounded, so what happens when you integrate on $[0,a]$ with $ato0$?
    – Jean-Claude Arbaut
    Nov 23 at 19:45










  • The integrand is continuous on [0,pi] considering $ f_n(0) $=1 and hence it is bounded on [0,pi]. Sorry I am a beginner in real analysis ..I could not understand what you said .
    – Normal
    Nov 23 at 20:17
















1












1








1







If $a>0,$ show that $lim limits_{n to infty} int_{ a}^{pi} frac{sin nx}{nx} mathrm dx =0,$ what happens if $a=0.$

The first part is easy since the integrand uniformly converges to zero on $[a, pi]$ and for each $n, ; f_n(x)= frac {sin nx}{nx}$ is continuous on $[a,pi].$

But for $a=0$ I don't understand how to proceed. I was thinking of using bounded convergence theorem but that required uniform boundedness of the integrand .
Please give me a hint. Thanks in advance.










share|cite|improve this question















If $a>0,$ show that $lim limits_{n to infty} int_{ a}^{pi} frac{sin nx}{nx} mathrm dx =0,$ what happens if $a=0.$

The first part is easy since the integrand uniformly converges to zero on $[a, pi]$ and for each $n, ; f_n(x)= frac {sin nx}{nx}$ is continuous on $[a,pi].$

But for $a=0$ I don't understand how to proceed. I was thinking of using bounded convergence theorem but that required uniform boundedness of the integrand .
Please give me a hint. Thanks in advance.







real-analysis






share|cite|improve this question















share|cite|improve this question













share|cite|improve this question




share|cite|improve this question








edited Nov 23 at 19:35

























asked Nov 23 at 19:28









Normal

647




647












  • is this what you aimed to write? Please check.
    – user376343
    Nov 23 at 19:34






  • 1




    Where is $a$ in the integral?
    – Will M.
    Nov 23 at 19:34












  • No, sorry, I fixed it
    – Normal
    Nov 23 at 19:38










  • The integrand is continuous and bounded, so what happens when you integrate on $[0,a]$ with $ato0$?
    – Jean-Claude Arbaut
    Nov 23 at 19:45










  • The integrand is continuous on [0,pi] considering $ f_n(0) $=1 and hence it is bounded on [0,pi]. Sorry I am a beginner in real analysis ..I could not understand what you said .
    – Normal
    Nov 23 at 20:17




















  • is this what you aimed to write? Please check.
    – user376343
    Nov 23 at 19:34






  • 1




    Where is $a$ in the integral?
    – Will M.
    Nov 23 at 19:34












  • No, sorry, I fixed it
    – Normal
    Nov 23 at 19:38










  • The integrand is continuous and bounded, so what happens when you integrate on $[0,a]$ with $ato0$?
    – Jean-Claude Arbaut
    Nov 23 at 19:45










  • The integrand is continuous on [0,pi] considering $ f_n(0) $=1 and hence it is bounded on [0,pi]. Sorry I am a beginner in real analysis ..I could not understand what you said .
    – Normal
    Nov 23 at 20:17


















is this what you aimed to write? Please check.
– user376343
Nov 23 at 19:34




is this what you aimed to write? Please check.
– user376343
Nov 23 at 19:34




1




1




Where is $a$ in the integral?
– Will M.
Nov 23 at 19:34






Where is $a$ in the integral?
– Will M.
Nov 23 at 19:34














No, sorry, I fixed it
– Normal
Nov 23 at 19:38




No, sorry, I fixed it
– Normal
Nov 23 at 19:38












The integrand is continuous and bounded, so what happens when you integrate on $[0,a]$ with $ato0$?
– Jean-Claude Arbaut
Nov 23 at 19:45




The integrand is continuous and bounded, so what happens when you integrate on $[0,a]$ with $ato0$?
– Jean-Claude Arbaut
Nov 23 at 19:45












The integrand is continuous on [0,pi] considering $ f_n(0) $=1 and hence it is bounded on [0,pi]. Sorry I am a beginner in real analysis ..I could not understand what you said .
– Normal
Nov 23 at 20:17






The integrand is continuous on [0,pi] considering $ f_n(0) $=1 and hence it is bounded on [0,pi]. Sorry I am a beginner in real analysis ..I could not understand what you said .
– Normal
Nov 23 at 20:17












2 Answers
2






active

oldest

votes


















1














For all $x$, $|f_n(x)|le1$, hence



$$left|int_0^{a}f_n(x)dxright|le (a-0) sup_{[0,a]}|f_n|le a$$



Therefore



$$left|int_0^pi f_n(x)dxright|=left|int_0^a f_n(x)dx+int_a^pi f_n(x)dxright|le a+left|int_a^pi f_n(x)dxright|$$



Now, for all $epsilon>0$, let $a=epsilon$, and the remaining integral term tends to $0$ as $nto0$ (as $a$ is fixed as soon as $epsilon$ is fixed), so there is a $N$ such that for all $n>N$, $left|int_a^pi f_n(x)dxright|<epsilon$.



Hence, for all $epsilon>0$, there is a $N$ such that for all $n>N$, $left|int_0^pi f_n(x)dxright|<2epsilon$.



Hence $int_0^pi f_n(x)dxto0$ as $ntoinfty$.






share|cite|improve this answer





















  • Thank you ..I have a question....first the given sequences of function are not defined at 0 but since the limit exists at zero and the limiting value is 1 , I can define $ f_n(0) $=1, so did you write $ | f_n(x)| le 1$ with the same thought I have or something else ..If there is an other explanation please tell me ....so then $ f_n(x) $ is uniformly bounded and I can use bounded convergence theorem to conclude that the limiting value is zero..can I do that ?
    – Normal
    Nov 23 at 22:19










  • @Normal Yes, you can define $f_n(0)=1$ because the limit exists. It's a removable singularity, and the function thus defined is $C^{infty}$. See also en.wikipedia.org/wiki/Sinc_function
    – Jean-Claude Arbaut
    Nov 23 at 22:21












  • Thank you... please tell me it will be ok if I use bounded convergence theorem by using the uniform boundedness of $ f_n(x) $
    – Normal
    Nov 23 at 22:38










  • @Normal Yes, it also works. But you're not really a beginner then :)
    – Jean-Claude Arbaut
    Nov 23 at 22:41










  • One more question what if I choose an $ epsilon $ different from a .
    – Normal
    Nov 23 at 23:05





















3














We have that $$int_{0}^{pi }frac{sin nx}{nx} mathrm dx=int_{0}^{npi }frac{sin nx}{nx} mathrm {1over n}dnx={1over n }int_{0}^{npi} {sin xover x}dx$$also we know that $$int_0^{infty}{sin xover x}dx={pi over 2}$$according to Integration of Sinc function therefore $$lim _{nto infty}int_{0}^{pi }frac{sin nx}{nx} mathrm dx=0$$






share|cite|improve this answer





















  • Thank you .So $ lim_{nto infty} int_{0}^{pi} frac{sin nx}{nx} $=$( lim_{nto infty} {1 over n})(lim_{nto infty} int_{0}^{ npi}{sin x over x}) dx $ =0×(pi/2)=0. Is it what you did?
    – Normal
    Nov 23 at 20:05










  • You're welcome. Also could be explained that way. Yes...
    – Mostafa Ayaz
    Nov 23 at 20:10












  • Sorry for disturbing you again ,I have a doubt am I allowed to write what I wrote above..I mean I know lim ($ x_n y_n $ ) = lim ($ x_n $)lim ( $ y_n $) when the both limit exists ..but am I allowed to do the same thing in here.
    – Normal
    Nov 23 at 21:18










  • No mind at all :) Notice that i concluded that $$0le lim int_{0}^{pi }frac{sin nx}{nx} mathrm dx= lim {1over n }int_{0}^{npi} {sin xover x}dxlelim {1over n }int_{0}^{infty} {sin xover x}dx=lim {pi over 2n}=0$$ and used squeeze theorem
    – Mostafa Ayaz
    Nov 23 at 21:21










  • Thanks for answering though I didn't understand the line where you wrote lim $ {1 over n} int_{0}^{npi} { sin x over x} dx le lim {1 over n} int_{0}^{infty} {sin x over x} dx $ but it's alright the function is uniformly bounded defining $ f_n(0) =1 $ so by bounded convergence theorem I can conclude the answer is zero .
    – Normal
    Nov 23 at 22:31













Your Answer





StackExchange.ifUsing("editor", function () {
return StackExchange.using("mathjaxEditing", function () {
StackExchange.MarkdownEditor.creationCallbacks.add(function (editor, postfix) {
StackExchange.mathjaxEditing.prepareWmdForMathJax(editor, postfix, [["$", "$"], ["\\(","\\)"]]);
});
});
}, "mathjax-editing");

StackExchange.ready(function() {
var channelOptions = {
tags: "".split(" "),
id: "69"
};
initTagRenderer("".split(" "), "".split(" "), channelOptions);

StackExchange.using("externalEditor", function() {
// Have to fire editor after snippets, if snippets enabled
if (StackExchange.settings.snippets.snippetsEnabled) {
StackExchange.using("snippets", function() {
createEditor();
});
}
else {
createEditor();
}
});

function createEditor() {
StackExchange.prepareEditor({
heartbeatType: 'answer',
autoActivateHeartbeat: false,
convertImagesToLinks: true,
noModals: true,
showLowRepImageUploadWarning: true,
reputationToPostImages: 10,
bindNavPrevention: true,
postfix: "",
imageUploader: {
brandingHtml: "Powered by u003ca class="icon-imgur-white" href="https://imgur.com/"u003eu003c/au003e",
contentPolicyHtml: "User contributions licensed under u003ca href="https://creativecommons.org/licenses/by-sa/3.0/"u003ecc by-sa 3.0 with attribution requiredu003c/au003e u003ca href="https://stackoverflow.com/legal/content-policy"u003e(content policy)u003c/au003e",
allowUrls: true
},
noCode: true, onDemand: true,
discardSelector: ".discard-answer"
,immediatelyShowMarkdownHelp:true
});


}
});














draft saved

draft discarded


















StackExchange.ready(
function () {
StackExchange.openid.initPostLogin('.new-post-login', 'https%3a%2f%2fmath.stackexchange.com%2fquestions%2f3010741%2ffind-the-limit-of-int-0-pi-frac-sin-nxnx-mathrm-dx%23new-answer', 'question_page');
}
);

Post as a guest















Required, but never shown

























2 Answers
2






active

oldest

votes








2 Answers
2






active

oldest

votes









active

oldest

votes






active

oldest

votes









1














For all $x$, $|f_n(x)|le1$, hence



$$left|int_0^{a}f_n(x)dxright|le (a-0) sup_{[0,a]}|f_n|le a$$



Therefore



$$left|int_0^pi f_n(x)dxright|=left|int_0^a f_n(x)dx+int_a^pi f_n(x)dxright|le a+left|int_a^pi f_n(x)dxright|$$



Now, for all $epsilon>0$, let $a=epsilon$, and the remaining integral term tends to $0$ as $nto0$ (as $a$ is fixed as soon as $epsilon$ is fixed), so there is a $N$ such that for all $n>N$, $left|int_a^pi f_n(x)dxright|<epsilon$.



Hence, for all $epsilon>0$, there is a $N$ such that for all $n>N$, $left|int_0^pi f_n(x)dxright|<2epsilon$.



Hence $int_0^pi f_n(x)dxto0$ as $ntoinfty$.






share|cite|improve this answer





















  • Thank you ..I have a question....first the given sequences of function are not defined at 0 but since the limit exists at zero and the limiting value is 1 , I can define $ f_n(0) $=1, so did you write $ | f_n(x)| le 1$ with the same thought I have or something else ..If there is an other explanation please tell me ....so then $ f_n(x) $ is uniformly bounded and I can use bounded convergence theorem to conclude that the limiting value is zero..can I do that ?
    – Normal
    Nov 23 at 22:19










  • @Normal Yes, you can define $f_n(0)=1$ because the limit exists. It's a removable singularity, and the function thus defined is $C^{infty}$. See also en.wikipedia.org/wiki/Sinc_function
    – Jean-Claude Arbaut
    Nov 23 at 22:21












  • Thank you... please tell me it will be ok if I use bounded convergence theorem by using the uniform boundedness of $ f_n(x) $
    – Normal
    Nov 23 at 22:38










  • @Normal Yes, it also works. But you're not really a beginner then :)
    – Jean-Claude Arbaut
    Nov 23 at 22:41










  • One more question what if I choose an $ epsilon $ different from a .
    – Normal
    Nov 23 at 23:05


















1














For all $x$, $|f_n(x)|le1$, hence



$$left|int_0^{a}f_n(x)dxright|le (a-0) sup_{[0,a]}|f_n|le a$$



Therefore



$$left|int_0^pi f_n(x)dxright|=left|int_0^a f_n(x)dx+int_a^pi f_n(x)dxright|le a+left|int_a^pi f_n(x)dxright|$$



Now, for all $epsilon>0$, let $a=epsilon$, and the remaining integral term tends to $0$ as $nto0$ (as $a$ is fixed as soon as $epsilon$ is fixed), so there is a $N$ such that for all $n>N$, $left|int_a^pi f_n(x)dxright|<epsilon$.



Hence, for all $epsilon>0$, there is a $N$ such that for all $n>N$, $left|int_0^pi f_n(x)dxright|<2epsilon$.



Hence $int_0^pi f_n(x)dxto0$ as $ntoinfty$.






share|cite|improve this answer





















  • Thank you ..I have a question....first the given sequences of function are not defined at 0 but since the limit exists at zero and the limiting value is 1 , I can define $ f_n(0) $=1, so did you write $ | f_n(x)| le 1$ with the same thought I have or something else ..If there is an other explanation please tell me ....so then $ f_n(x) $ is uniformly bounded and I can use bounded convergence theorem to conclude that the limiting value is zero..can I do that ?
    – Normal
    Nov 23 at 22:19










  • @Normal Yes, you can define $f_n(0)=1$ because the limit exists. It's a removable singularity, and the function thus defined is $C^{infty}$. See also en.wikipedia.org/wiki/Sinc_function
    – Jean-Claude Arbaut
    Nov 23 at 22:21












  • Thank you... please tell me it will be ok if I use bounded convergence theorem by using the uniform boundedness of $ f_n(x) $
    – Normal
    Nov 23 at 22:38










  • @Normal Yes, it also works. But you're not really a beginner then :)
    – Jean-Claude Arbaut
    Nov 23 at 22:41










  • One more question what if I choose an $ epsilon $ different from a .
    – Normal
    Nov 23 at 23:05
















1












1








1






For all $x$, $|f_n(x)|le1$, hence



$$left|int_0^{a}f_n(x)dxright|le (a-0) sup_{[0,a]}|f_n|le a$$



Therefore



$$left|int_0^pi f_n(x)dxright|=left|int_0^a f_n(x)dx+int_a^pi f_n(x)dxright|le a+left|int_a^pi f_n(x)dxright|$$



Now, for all $epsilon>0$, let $a=epsilon$, and the remaining integral term tends to $0$ as $nto0$ (as $a$ is fixed as soon as $epsilon$ is fixed), so there is a $N$ such that for all $n>N$, $left|int_a^pi f_n(x)dxright|<epsilon$.



Hence, for all $epsilon>0$, there is a $N$ such that for all $n>N$, $left|int_0^pi f_n(x)dxright|<2epsilon$.



Hence $int_0^pi f_n(x)dxto0$ as $ntoinfty$.






share|cite|improve this answer












For all $x$, $|f_n(x)|le1$, hence



$$left|int_0^{a}f_n(x)dxright|le (a-0) sup_{[0,a]}|f_n|le a$$



Therefore



$$left|int_0^pi f_n(x)dxright|=left|int_0^a f_n(x)dx+int_a^pi f_n(x)dxright|le a+left|int_a^pi f_n(x)dxright|$$



Now, for all $epsilon>0$, let $a=epsilon$, and the remaining integral term tends to $0$ as $nto0$ (as $a$ is fixed as soon as $epsilon$ is fixed), so there is a $N$ such that for all $n>N$, $left|int_a^pi f_n(x)dxright|<epsilon$.



Hence, for all $epsilon>0$, there is a $N$ such that for all $n>N$, $left|int_0^pi f_n(x)dxright|<2epsilon$.



Hence $int_0^pi f_n(x)dxto0$ as $ntoinfty$.







share|cite|improve this answer












share|cite|improve this answer



share|cite|improve this answer










answered Nov 23 at 21:15









Jean-Claude Arbaut

14.7k63464




14.7k63464












  • Thank you ..I have a question....first the given sequences of function are not defined at 0 but since the limit exists at zero and the limiting value is 1 , I can define $ f_n(0) $=1, so did you write $ | f_n(x)| le 1$ with the same thought I have or something else ..If there is an other explanation please tell me ....so then $ f_n(x) $ is uniformly bounded and I can use bounded convergence theorem to conclude that the limiting value is zero..can I do that ?
    – Normal
    Nov 23 at 22:19










  • @Normal Yes, you can define $f_n(0)=1$ because the limit exists. It's a removable singularity, and the function thus defined is $C^{infty}$. See also en.wikipedia.org/wiki/Sinc_function
    – Jean-Claude Arbaut
    Nov 23 at 22:21












  • Thank you... please tell me it will be ok if I use bounded convergence theorem by using the uniform boundedness of $ f_n(x) $
    – Normal
    Nov 23 at 22:38










  • @Normal Yes, it also works. But you're not really a beginner then :)
    – Jean-Claude Arbaut
    Nov 23 at 22:41










  • One more question what if I choose an $ epsilon $ different from a .
    – Normal
    Nov 23 at 23:05




















  • Thank you ..I have a question....first the given sequences of function are not defined at 0 but since the limit exists at zero and the limiting value is 1 , I can define $ f_n(0) $=1, so did you write $ | f_n(x)| le 1$ with the same thought I have or something else ..If there is an other explanation please tell me ....so then $ f_n(x) $ is uniformly bounded and I can use bounded convergence theorem to conclude that the limiting value is zero..can I do that ?
    – Normal
    Nov 23 at 22:19










  • @Normal Yes, you can define $f_n(0)=1$ because the limit exists. It's a removable singularity, and the function thus defined is $C^{infty}$. See also en.wikipedia.org/wiki/Sinc_function
    – Jean-Claude Arbaut
    Nov 23 at 22:21












  • Thank you... please tell me it will be ok if I use bounded convergence theorem by using the uniform boundedness of $ f_n(x) $
    – Normal
    Nov 23 at 22:38










  • @Normal Yes, it also works. But you're not really a beginner then :)
    – Jean-Claude Arbaut
    Nov 23 at 22:41










  • One more question what if I choose an $ epsilon $ different from a .
    – Normal
    Nov 23 at 23:05


















Thank you ..I have a question....first the given sequences of function are not defined at 0 but since the limit exists at zero and the limiting value is 1 , I can define $ f_n(0) $=1, so did you write $ | f_n(x)| le 1$ with the same thought I have or something else ..If there is an other explanation please tell me ....so then $ f_n(x) $ is uniformly bounded and I can use bounded convergence theorem to conclude that the limiting value is zero..can I do that ?
– Normal
Nov 23 at 22:19




Thank you ..I have a question....first the given sequences of function are not defined at 0 but since the limit exists at zero and the limiting value is 1 , I can define $ f_n(0) $=1, so did you write $ | f_n(x)| le 1$ with the same thought I have or something else ..If there is an other explanation please tell me ....so then $ f_n(x) $ is uniformly bounded and I can use bounded convergence theorem to conclude that the limiting value is zero..can I do that ?
– Normal
Nov 23 at 22:19












@Normal Yes, you can define $f_n(0)=1$ because the limit exists. It's a removable singularity, and the function thus defined is $C^{infty}$. See also en.wikipedia.org/wiki/Sinc_function
– Jean-Claude Arbaut
Nov 23 at 22:21






@Normal Yes, you can define $f_n(0)=1$ because the limit exists. It's a removable singularity, and the function thus defined is $C^{infty}$. See also en.wikipedia.org/wiki/Sinc_function
– Jean-Claude Arbaut
Nov 23 at 22:21














Thank you... please tell me it will be ok if I use bounded convergence theorem by using the uniform boundedness of $ f_n(x) $
– Normal
Nov 23 at 22:38




Thank you... please tell me it will be ok if I use bounded convergence theorem by using the uniform boundedness of $ f_n(x) $
– Normal
Nov 23 at 22:38












@Normal Yes, it also works. But you're not really a beginner then :)
– Jean-Claude Arbaut
Nov 23 at 22:41




@Normal Yes, it also works. But you're not really a beginner then :)
– Jean-Claude Arbaut
Nov 23 at 22:41












One more question what if I choose an $ epsilon $ different from a .
– Normal
Nov 23 at 23:05






One more question what if I choose an $ epsilon $ different from a .
– Normal
Nov 23 at 23:05













3














We have that $$int_{0}^{pi }frac{sin nx}{nx} mathrm dx=int_{0}^{npi }frac{sin nx}{nx} mathrm {1over n}dnx={1over n }int_{0}^{npi} {sin xover x}dx$$also we know that $$int_0^{infty}{sin xover x}dx={pi over 2}$$according to Integration of Sinc function therefore $$lim _{nto infty}int_{0}^{pi }frac{sin nx}{nx} mathrm dx=0$$






share|cite|improve this answer





















  • Thank you .So $ lim_{nto infty} int_{0}^{pi} frac{sin nx}{nx} $=$( lim_{nto infty} {1 over n})(lim_{nto infty} int_{0}^{ npi}{sin x over x}) dx $ =0×(pi/2)=0. Is it what you did?
    – Normal
    Nov 23 at 20:05










  • You're welcome. Also could be explained that way. Yes...
    – Mostafa Ayaz
    Nov 23 at 20:10












  • Sorry for disturbing you again ,I have a doubt am I allowed to write what I wrote above..I mean I know lim ($ x_n y_n $ ) = lim ($ x_n $)lim ( $ y_n $) when the both limit exists ..but am I allowed to do the same thing in here.
    – Normal
    Nov 23 at 21:18










  • No mind at all :) Notice that i concluded that $$0le lim int_{0}^{pi }frac{sin nx}{nx} mathrm dx= lim {1over n }int_{0}^{npi} {sin xover x}dxlelim {1over n }int_{0}^{infty} {sin xover x}dx=lim {pi over 2n}=0$$ and used squeeze theorem
    – Mostafa Ayaz
    Nov 23 at 21:21










  • Thanks for answering though I didn't understand the line where you wrote lim $ {1 over n} int_{0}^{npi} { sin x over x} dx le lim {1 over n} int_{0}^{infty} {sin x over x} dx $ but it's alright the function is uniformly bounded defining $ f_n(0) =1 $ so by bounded convergence theorem I can conclude the answer is zero .
    – Normal
    Nov 23 at 22:31


















3














We have that $$int_{0}^{pi }frac{sin nx}{nx} mathrm dx=int_{0}^{npi }frac{sin nx}{nx} mathrm {1over n}dnx={1over n }int_{0}^{npi} {sin xover x}dx$$also we know that $$int_0^{infty}{sin xover x}dx={pi over 2}$$according to Integration of Sinc function therefore $$lim _{nto infty}int_{0}^{pi }frac{sin nx}{nx} mathrm dx=0$$






share|cite|improve this answer





















  • Thank you .So $ lim_{nto infty} int_{0}^{pi} frac{sin nx}{nx} $=$( lim_{nto infty} {1 over n})(lim_{nto infty} int_{0}^{ npi}{sin x over x}) dx $ =0×(pi/2)=0. Is it what you did?
    – Normal
    Nov 23 at 20:05










  • You're welcome. Also could be explained that way. Yes...
    – Mostafa Ayaz
    Nov 23 at 20:10












  • Sorry for disturbing you again ,I have a doubt am I allowed to write what I wrote above..I mean I know lim ($ x_n y_n $ ) = lim ($ x_n $)lim ( $ y_n $) when the both limit exists ..but am I allowed to do the same thing in here.
    – Normal
    Nov 23 at 21:18










  • No mind at all :) Notice that i concluded that $$0le lim int_{0}^{pi }frac{sin nx}{nx} mathrm dx= lim {1over n }int_{0}^{npi} {sin xover x}dxlelim {1over n }int_{0}^{infty} {sin xover x}dx=lim {pi over 2n}=0$$ and used squeeze theorem
    – Mostafa Ayaz
    Nov 23 at 21:21










  • Thanks for answering though I didn't understand the line where you wrote lim $ {1 over n} int_{0}^{npi} { sin x over x} dx le lim {1 over n} int_{0}^{infty} {sin x over x} dx $ but it's alright the function is uniformly bounded defining $ f_n(0) =1 $ so by bounded convergence theorem I can conclude the answer is zero .
    – Normal
    Nov 23 at 22:31
















3












3








3






We have that $$int_{0}^{pi }frac{sin nx}{nx} mathrm dx=int_{0}^{npi }frac{sin nx}{nx} mathrm {1over n}dnx={1over n }int_{0}^{npi} {sin xover x}dx$$also we know that $$int_0^{infty}{sin xover x}dx={pi over 2}$$according to Integration of Sinc function therefore $$lim _{nto infty}int_{0}^{pi }frac{sin nx}{nx} mathrm dx=0$$






share|cite|improve this answer












We have that $$int_{0}^{pi }frac{sin nx}{nx} mathrm dx=int_{0}^{npi }frac{sin nx}{nx} mathrm {1over n}dnx={1over n }int_{0}^{npi} {sin xover x}dx$$also we know that $$int_0^{infty}{sin xover x}dx={pi over 2}$$according to Integration of Sinc function therefore $$lim _{nto infty}int_{0}^{pi }frac{sin nx}{nx} mathrm dx=0$$







share|cite|improve this answer












share|cite|improve this answer



share|cite|improve this answer










answered Nov 23 at 19:43









Mostafa Ayaz

13.7k3836




13.7k3836












  • Thank you .So $ lim_{nto infty} int_{0}^{pi} frac{sin nx}{nx} $=$( lim_{nto infty} {1 over n})(lim_{nto infty} int_{0}^{ npi}{sin x over x}) dx $ =0×(pi/2)=0. Is it what you did?
    – Normal
    Nov 23 at 20:05










  • You're welcome. Also could be explained that way. Yes...
    – Mostafa Ayaz
    Nov 23 at 20:10












  • Sorry for disturbing you again ,I have a doubt am I allowed to write what I wrote above..I mean I know lim ($ x_n y_n $ ) = lim ($ x_n $)lim ( $ y_n $) when the both limit exists ..but am I allowed to do the same thing in here.
    – Normal
    Nov 23 at 21:18










  • No mind at all :) Notice that i concluded that $$0le lim int_{0}^{pi }frac{sin nx}{nx} mathrm dx= lim {1over n }int_{0}^{npi} {sin xover x}dxlelim {1over n }int_{0}^{infty} {sin xover x}dx=lim {pi over 2n}=0$$ and used squeeze theorem
    – Mostafa Ayaz
    Nov 23 at 21:21










  • Thanks for answering though I didn't understand the line where you wrote lim $ {1 over n} int_{0}^{npi} { sin x over x} dx le lim {1 over n} int_{0}^{infty} {sin x over x} dx $ but it's alright the function is uniformly bounded defining $ f_n(0) =1 $ so by bounded convergence theorem I can conclude the answer is zero .
    – Normal
    Nov 23 at 22:31




















  • Thank you .So $ lim_{nto infty} int_{0}^{pi} frac{sin nx}{nx} $=$( lim_{nto infty} {1 over n})(lim_{nto infty} int_{0}^{ npi}{sin x over x}) dx $ =0×(pi/2)=0. Is it what you did?
    – Normal
    Nov 23 at 20:05










  • You're welcome. Also could be explained that way. Yes...
    – Mostafa Ayaz
    Nov 23 at 20:10












  • Sorry for disturbing you again ,I have a doubt am I allowed to write what I wrote above..I mean I know lim ($ x_n y_n $ ) = lim ($ x_n $)lim ( $ y_n $) when the both limit exists ..but am I allowed to do the same thing in here.
    – Normal
    Nov 23 at 21:18










  • No mind at all :) Notice that i concluded that $$0le lim int_{0}^{pi }frac{sin nx}{nx} mathrm dx= lim {1over n }int_{0}^{npi} {sin xover x}dxlelim {1over n }int_{0}^{infty} {sin xover x}dx=lim {pi over 2n}=0$$ and used squeeze theorem
    – Mostafa Ayaz
    Nov 23 at 21:21










  • Thanks for answering though I didn't understand the line where you wrote lim $ {1 over n} int_{0}^{npi} { sin x over x} dx le lim {1 over n} int_{0}^{infty} {sin x over x} dx $ but it's alright the function is uniformly bounded defining $ f_n(0) =1 $ so by bounded convergence theorem I can conclude the answer is zero .
    – Normal
    Nov 23 at 22:31


















Thank you .So $ lim_{nto infty} int_{0}^{pi} frac{sin nx}{nx} $=$( lim_{nto infty} {1 over n})(lim_{nto infty} int_{0}^{ npi}{sin x over x}) dx $ =0×(pi/2)=0. Is it what you did?
– Normal
Nov 23 at 20:05




Thank you .So $ lim_{nto infty} int_{0}^{pi} frac{sin nx}{nx} $=$( lim_{nto infty} {1 over n})(lim_{nto infty} int_{0}^{ npi}{sin x over x}) dx $ =0×(pi/2)=0. Is it what you did?
– Normal
Nov 23 at 20:05












You're welcome. Also could be explained that way. Yes...
– Mostafa Ayaz
Nov 23 at 20:10






You're welcome. Also could be explained that way. Yes...
– Mostafa Ayaz
Nov 23 at 20:10














Sorry for disturbing you again ,I have a doubt am I allowed to write what I wrote above..I mean I know lim ($ x_n y_n $ ) = lim ($ x_n $)lim ( $ y_n $) when the both limit exists ..but am I allowed to do the same thing in here.
– Normal
Nov 23 at 21:18




Sorry for disturbing you again ,I have a doubt am I allowed to write what I wrote above..I mean I know lim ($ x_n y_n $ ) = lim ($ x_n $)lim ( $ y_n $) when the both limit exists ..but am I allowed to do the same thing in here.
– Normal
Nov 23 at 21:18












No mind at all :) Notice that i concluded that $$0le lim int_{0}^{pi }frac{sin nx}{nx} mathrm dx= lim {1over n }int_{0}^{npi} {sin xover x}dxlelim {1over n }int_{0}^{infty} {sin xover x}dx=lim {pi over 2n}=0$$ and used squeeze theorem
– Mostafa Ayaz
Nov 23 at 21:21




No mind at all :) Notice that i concluded that $$0le lim int_{0}^{pi }frac{sin nx}{nx} mathrm dx= lim {1over n }int_{0}^{npi} {sin xover x}dxlelim {1over n }int_{0}^{infty} {sin xover x}dx=lim {pi over 2n}=0$$ and used squeeze theorem
– Mostafa Ayaz
Nov 23 at 21:21












Thanks for answering though I didn't understand the line where you wrote lim $ {1 over n} int_{0}^{npi} { sin x over x} dx le lim {1 over n} int_{0}^{infty} {sin x over x} dx $ but it's alright the function is uniformly bounded defining $ f_n(0) =1 $ so by bounded convergence theorem I can conclude the answer is zero .
– Normal
Nov 23 at 22:31






Thanks for answering though I didn't understand the line where you wrote lim $ {1 over n} int_{0}^{npi} { sin x over x} dx le lim {1 over n} int_{0}^{infty} {sin x over x} dx $ but it's alright the function is uniformly bounded defining $ f_n(0) =1 $ so by bounded convergence theorem I can conclude the answer is zero .
– Normal
Nov 23 at 22:31




















draft saved

draft discarded




















































Thanks for contributing an answer to Mathematics Stack Exchange!


  • Please be sure to answer the question. Provide details and share your research!

But avoid



  • Asking for help, clarification, or responding to other answers.

  • Making statements based on opinion; back them up with references or personal experience.


Use MathJax to format equations. MathJax reference.


To learn more, see our tips on writing great answers.





Some of your past answers have not been well-received, and you're in danger of being blocked from answering.


Please pay close attention to the following guidance:


  • Please be sure to answer the question. Provide details and share your research!

But avoid



  • Asking for help, clarification, or responding to other answers.

  • Making statements based on opinion; back them up with references or personal experience.


To learn more, see our tips on writing great answers.




draft saved


draft discarded














StackExchange.ready(
function () {
StackExchange.openid.initPostLogin('.new-post-login', 'https%3a%2f%2fmath.stackexchange.com%2fquestions%2f3010741%2ffind-the-limit-of-int-0-pi-frac-sin-nxnx-mathrm-dx%23new-answer', 'question_page');
}
);

Post as a guest















Required, but never shown





















































Required, but never shown














Required, but never shown












Required, but never shown







Required, but never shown

































Required, but never shown














Required, but never shown












Required, but never shown







Required, but never shown







Popular posts from this blog

Plaza Victoria

In PowerPoint, is there a keyboard shortcut for bulleted / numbered list?

How to put 3 figures in Latex with 2 figures side by side and 1 below these side by side images but in...